If two vertices of an equilateral triangle are \( \Large \left(0,\ 0\right)\ and\ \left(3,\ 3\sqrt{3}\right) \) then the third vertex is:

A) \( \Large \left(3,\ -3\right) \)

B) \( \Large \left(-3,\ 3\right) \)

C) \( \Large \left(-3,\ 3\sqrt{3}\right) \)

D) none of these

Correct answer:
C) \( \Large \left(-3,\ 3\sqrt{3}\right) \)

Description for Correct answer:
Angle AOM is \( \Large 30 ^{\circ} \). Hence required point of B is \( \Large \left(-3,\ 3\sqrt{3}\right) \)



Please provide the error details in above question